0
$\begingroup$

Claim 1: If $f: [a, b] \rightarrow \mathbb{R}$ is continuous, then $f$ assumes a maximum value

I know there's a theorem that states if $f$ is a continuous real-valued function on a closed interval $[a, b]$, then $f$ is a bounded function. And $f$ assumes its max and min values on $[a, b]$.

Going back to the claim, it doesn't state that $f$ is necessarily bounded. (I understood it as $x$ are bounded, i.e., $a \leq x \leq b$, but $f(x)$ is not necessarily bounded? Since $f(x) \in \mathbb{R}$, for $a \leq x \leq b$). So I think it is false. And my counterexample is $f(x) = tan(x)$ where $-\frac{\pi}{2} \leq x \leq \frac{\pi}{2}$. Although $f: [a, b] \rightarrow \mathbb{R}$ is continuous, it is not bounded, so it does not assume a maximum value. Is my logic correct?

Claim 2: If $f: [a, b] \rightarrow \mathbb{R}$ assumes a maximum, then $f$ is continuous.

I feel like this is not necessarily true, either? Since a function might have a jump discontinuity and that point might be the max, but the function is not continuous. Would this be correct?

$\endgroup$
4
  • 1
    $\begingroup$ Your $f$ is not continuous, nor even defined, on all of $[-\pi/2,\pi/2]$. $\endgroup$ Commented Jan 13, 2015 at 22:19
  • $\begingroup$ $f(x) = \tan x$ is not continuous at $x = \pm \frac{\pi}{2}$ and claim 2 as you point out is not necessarily true, $f$ can have discontinuity at a point and still have maxima and minima in its domain of definition. $\endgroup$ Commented Jan 13, 2015 at 22:19
  • $\begingroup$ I see. So claim 1 is true then? $\endgroup$ Commented Jan 13, 2015 at 22:22
  • $\begingroup$ Claim 1 is true. Claim 2 clearly not. Just take indicator functions to get a counter example. By the way: A continuous function on a compact interval, as $[a,b]$ is, must also be bounded. $\endgroup$ Commented Jan 13, 2015 at 22:25

1 Answer 1

0
$\begingroup$

Claim 1 is the Extreme value theorem by Weierstrass.

Claim 2 is false.

Easy example:

$$ f [0,1] \to \mathbb{R} , f(x) = \begin{cases} x & x \neq 1 \\ 10 & x=1 \\ \end{cases} $$

has a maximum ( $\max(f)=10$ ), but is not continous in $x=1$

$\endgroup$

You must log in to answer this question.

Start asking to get answers

Find the answer to your question by asking.

Ask question

Explore related questions

See similar questions with these tags.